Let f(t) be the temperature (in degrees Celsius) of a liquid at time t (in hours). The rate of temperature change at time a has the value f(a). Determine the proper method of solution for the question.By how many degrees did the temperature rise during the first 4 hours?Which of the following will result in the number of degrees the temperature of the liquid rose during the first 4 hours?OA Compute f'(4).OB. Compute 1(4).OC. Subtract the liquid's initial temperature from its temperature 4 hours later.OD. Subtract the liquid's initial temperature from its temperature 4 hours later and divide by 4.

Answers

Answer 1

The proper method of solution for the question "By how many degrees did the temperature rise during the first 4 hours?" is to subtract the liquid's initial temperature from its temperature 4 hours later, which is option (C).

To find the change in temperature, we need to calculate the temperature difference between the initial and final temperatures of the liquid. Since we are asked about the temperature rise, we need to subtract the initial temperature from the temperature after 4 hours. This gives us the total increase in temperature. Option (A) is incorrect because it only gives the value of the rate of change of temperature at time 4, but not the temperature change over the entire 4 hour period. Option (B) is also incorrect, as it does not provide any information about the temperature at all. Option (D) is incorrect because dividing by 4 assumes that the temperature change is constant over the entire 4 hour period, which may not be true. Therefore, option (C) is the correct method of solution to find the number of degrees the temperature of the liquid rose during the first 4 hours.

Learn more about temperature here

https://brainly.com/question/26866637

#SPJ11


Related Questions

The number 6545 can be written as a product of a pair of positive two digit integers. What are these two integers?

Answers

Answer:

77 and 85

Step-by-step explanation:

Since the number 6545 ends in a 5, we can assume that one of the two digit numbers ends in a 5. I tried 55, 65, 75, and 85, and I just divided 6545 by those numbers until I got a two-digit positive integer as my answer,

85 * 77 = 6545

Therefore, the answer is 85 and 77.

I hope this helps!

Given g(x) = -5x+ 2, find g(4).
Answer:

Answers

Answer:

-18

Step-by-step explanation:

g(x)=-5x+2

g(4)=-5(4)+2=-20+2=-18

Answer:

-18

Step-by-step explanation:

The sum of two numbers is 12. The product of those same two numbers is -64. Find the numbers.​

Answers

Answer: The answer for the 2 numbers are 16 and -4.

Step-by-step explanation: 16 + (-4) is 12, and 16 x (-4) is -64.

-2 + b < 3





.............

Answers

Answer:

I believe we are solving for b. b<5

Step-by-step explanation:

this is just for fun ong sooo yeaaa **delete spaces**
https://meet. go ogle. com/vqk-wfjs-dsa

Answers

Answer:

im fanna join      

Step-by-step explanation:

okayy lolkajaja abahahahajajjsjdhdhd

Answer the photo below thanks

Answers

Answer:

You're right, the answer is C.

Step-by-step explanation:

Answer:

C

Step-by-step explanation:

I am not sure if it is B or C, but I can say that it is not A since the figure in A is rotated. Figure in B kinda looks like a reflection then C because if the figures were translated then the corner should be at one side. Sorry if that kinda confused you, but I added a picture of what I am trying to say for the figure in option B. I guess it is C.

Hope this helps, thank you !!

3÷1 1/4
i need to know ASAP and i need to know how to do it

Answers

Answer:

2.4 or 2[tex]\frac{2}{5}[/tex]

Step-by-step explanation:

Let's set this up:

We have 3/1[tex]\frac{1}{4}[/tex]

That's also 3/1.25

For division, we don't divide if there is a fraction, so we move the decimal place over to the right for both the numbers

300/125

Then you long divide as normal:

And it's really hard to show how to long divide on a computer so I'm just going to go ahead and assume you know how to.

But the answer to that would be 2.4 or 2[tex]\frac{2}{5}[/tex]

Thank you, let me know if you have any questions.

It WILL BE 12/5

3 is 12/4

1 1/4 is 5/4

When you set it up like this 12/4 * 4/5 you can cancel out the 4’s and get 12/5 as the final answer.

Which of the following expressions are equivalent to 5(m + 3)? hurryy

Answers

Answer:

B   5m + 15

Step-by-step explanation:

Use the distributive property and multiply 5 by both terms inside the parentheses.

5(m + 3) = 5 * m + 5 * 3 = 5m + 15

Answer: B   5m + 15

finding value of x. PQR
P- 2x°
3
3
3

Answers

[tex]\mathfrak{\huge{\orange{\underline{\underline{AnSwEr:-}}}}}[/tex]

Actually Welcome to the Concept of the Equilateral Triangles.

Since, we know that, all the sides of a triangle are equal, hence it is a Equilateral triangle.

Equilateral triangle has one all angles equal to 60°.

so, major angle P = 60°

hence, 2x=60°

===> x = 30°

A. Linear
B. Exponential
C. Neither

Answers

the answer is C. neither

7/8m^3 +3-1/2m^3. can u please help me solve this

Answers

Answer:

3m^3/8 + 3

Step-by-step explanation:

It can't be solved but that's the most simplified version.

Answer:

15/8m^3

Step-by-step explanation:

7/8m^3+2/2m^3

7/8m^3+1/m^3

=15/8m^3

NEED HELP ASAPPPPPP PLZZZZZ
Right one paragraph on
What is factoring by using GCF?

Answers

Answer:

GCF is the largest factor that all terms have in common.

A package weighs 32 ounces. What is the weight of the package in pounds

Answers

Answer:

2 pounds

Step-by-step explanation:

1 pound=16 ounces

If you are making broccoli cheese soup the ratio of cheese to broccoli to broth is 5:6:10 if there is 100 parts broth how many of each ingredient should you use

Answers

Answer: See explanation

Step-by-step explanation:

Since there are 100 parts broth, the number to use for each ingredients will be:

Cheese = 5 × 100/10 = 5 × 10 = 50

Broccoli = 6 × 100/10 = 6 × 10 = 60

Broth = 100

the supplement of a 65° angle has a measure of​

Answers

Answer:

115 degrees

Step-by-step explanation:

supplementary angles add up to 180 degrees, so if x is the angle you're trying to have you get:

[tex]x + 65 = 180\\x = 115[/tex]

Order the following from LEAST to GREATEST: 3/5, -1/2, 0.50, -7/10, -0.75 *

Answers

Answer:

-0.75, -7/10, -1/2, 0.50, 3/5

Answer:

-0.75, -7/10, -1/2, 0.50, 3/5

Step-by-step explanation:

put all of them as a decimal first

3/5 = 0.6 (5)

-1/2 = -0.50 (3)

0.50 = 0.50 (4)

-7/10 = -0.70 (2)

-0.75 = -0.75 (1)

Which shows the distributive property?

1. 5x + 2 = 2 + 5x

2. 5(x + 2) = 5x + 10

3. 5(x + 2) = 5(2 + x)

Answers

Number 2 because your multiplying the 5 by the numbers in the parenthesis.

A water rocket is launched froni a platform. The height,
h in metres, of the water rocket at time & seconds after
it is launched is h= -212 + 74 + 4. When does the water
rocket hit the ground?​

Answers

Answer:

4.41seconds later

Step-by-step explanation:

Let the equation of the height reached by the water front be h = -2t^2+7t+4

The water hits the ground at h =0

Substitute

0 =  -2t^2+7t+4

-2t^2+7t+4 = 0

2t^2 - 7t - 8 = 0

t = 7±√(-7)²-4(2)(-8)/2(2)

t = 7±√49+64/4

t = 7±√113/4

t = 7±10.63/4

t = 7+10.63/4

t = 17.3/4

t = 4.41s

Hence the water hit the ground 4.41seconda later

2. it's not B
Write a rule for the linear function shown in the graph.



A. y = -5/2x - 1

B. y = 3x – 1

C. y = 2/5x + 1

D. y = 5/3x + 1

Answers

Answer:

Solve for the first variable in one of the equations, then substitute the result into the other equation.

Point Form:

( 0 ,   1 )

Equation Form:

x = 0 ,     y = 1

Step-by-step explanation:

Graph.

y = − 5/ 2 x − 1

y = 3 x − 1

y = 2/ 5 x + 1

y = 5/ 3 x + 1

PLEASE HELP!!! ILL GIVE BRAINLIEST, EXPLAIN YOUR ANSWER PLEASE

Answers

Answer: -20v^3 - 30x^2 +20

Step-by-step explanation:

simply multiply each term inside of the parentheses by -5.

Answer:

20v³ + 20 - 30x²

Step-by-step explanation:

multiply -5 by all three terms inside parentheses

(-5)(-4v³) + (-5)(-4) + (-5)(6x²)

20v³ + 20 - 30x²

Pls help me with this math problem

Answers

Answer:

12 ima shoot u

Step-by-step explanation:

cuántos encuentros son necesarios para una entrevista completa​

Answers

4 encuentros,creo Ó 2 encuentros.Pero yo creo que 4

Which values of a and b make the following equation true?

Answers

Answer:

a=11 , b=7

Step-by-step explanation:

When integer multiply with integer, the base were same, then we just add both powers.

for example

[tex]x {}^{2} \times {x}^{3} = {x}^{2 + 3} = {x}^{5} [/tex]

9. Teresa was taking her two children to the zoo. She purchased 1 adult ticket for herself and 2 children's
tickets for a total of $18.20. A family of 5 was also visiting the zoo. -The family purchased 2 adult tickets and
3 children's tickets for a total of $30.90. How much money did an adult ticket cost?

Answers

Answer:

plz mark as brainliest

Step-by-step explanation:

$7.9

1) Resolva as expressões numéricas. Não esqueça de deixar os cálculos.
a) 2 + 8-3-5 + 15 =
b) 12 + (35 - (10 + 2) +2] =
c) [(18+32) + 8 +5 3] + 6 =
d) 37 + (-25 - (-11 + 19 - 4)] =
e) 60 = {2 • (-7 + 18 = (-3 + 12)]} - [7 • (-3) - 18 = (-2) + 1) =
f) -8 +{-5 + [(8-12) + (13 + 12)] - 10) =​

Answers

Answer:

a=17

b=37

c=117

d=8

e=

f=-2

Given that ABCD is a parallelogram. Find the following

Answers

Answer:

m=8

m<A=83

m<D=97

Step-by-step explanation:

Angles that are opposite of each other in a parallelogram are equal. Knowing this, we can first conclude that angle A and angle C are equal. We can set up the following equation:

[tex]9m+11=8m+19[/tex]

Subtract both sides by 11

[tex]9m+11-11=8m+19-11\\9m=8m+8[/tex]

Subtract both sides by 8m

[tex]9m-8m=8m+8-8m\\m=8[/tex]

To calculate angle A, plug the value of m into its expression.

[tex]<A=9m+11\\<A=9(8)+11\\<A=72+11\\<A=83[/tex]

In a parallelogram, angles that share a side are supplementary, meaning they add up to 180 degrees. Therefore, angle A and angle D are supplementary. Knowing this, we can set up the following equation:

[tex]180=83+D[/tex]

Subtract both sides by 83

[tex]180-83=83+D-83\\97=D[/tex]

I hope this helps!

4(x - 9) plz answer quickly

Answers

Answer:

4x - 36

Step-by-step explanation:

B. Draw any pictures showing the ratios of:
1. 2 to 5
2.6 to 8
3.4 to 7
4.3 to 9​

Answers

this is for all of them

the output is double the input

Answers

Whats the question???

A local little league has a total of 60 ​players, of whom 40​% are left​-handed. How many left ​- handed players are​ there?

Answers

Answer:

24 players

Step-by-step explanation:

Answer:

24. 60 X 4/10 is 24

Step-by-step explanation:

Other Questions
Help pls Im in trouble so help John lockes perspective on the purpose of government is a _____. Which of the following tables represent a proportional relationship? which function below has a great rate of change a rain-slick surface Is the following statement true or false?All cells have the same DNA, but different types of cells express different sets of genes. Madison was reading an article on her phone that discussed macro and micronutrients. She learned that macronutrients provide energy to the body. Which of the following are macronutrients that the body needs? a. protein, minerals, and water b. vitamins, carbohydrates, and minerals c. carbohydrates, fats, and proteins d. fats, minerals, and protein e. vitamins, fats, and minerals Which of the following is a main reason why some states were not in favor of ratifying the US Constitution? A pet aardvark that regularly sells for $125.70 is on sale for 30% off. Find the sale price of the aardvark someone please tell me i only got 10 mins for this 5. Does John Legend believe his talent has improved or stayed the same? Has he done anything different since he became a singer?6. Define the word "cultivate"7. Is it normal to have both a fixed and growth mindset? 8. Change this sentence using the "Power of Yet" strategy: "I am not good at taking math tests."9. Tell me (2) situations when you have a fixed mindset shows up or when you've had a fixed mindset in the past. Write 2-3 sentences for EACH situation explaining the situation and why you think this mindset may show up. 10. Tell me how you can change your thinking about these situations to have a growth mindset. Write a growth mindset example you an say to yourself to overcome these situations. Write an example for EACH situation.NEED IT BY FRIDAY, PLS HELP ME AI's Auto Shop charges $149 for parts and $53 per hour of labor for a repair job. Edna's Engine Repair charges a flat fee of $280 for the same job. a. Write a system of equations to represent the situation. b. Graph the system of equations. c. Which shop is cheaper if the job takes 2 hours? 3 hours? A concave mirror has a focal length of 18 cm. This mirror forms an image located 54 cm in front of the mirror. What is the magnification of the mirror? (Include the sign.) A Physical Education (gym) class is playing a game of soccer. During the game, the ball experiences a net force of 0 newtons. Which statement explains what is most likely happening with the ball? Describe the benefits and limitations of digital connection Nine times the sum of a number and 8 is equal to 4 What is the plot structure of Animal Farm? i saw accepted the notion that conscience was no longer a private matter but one of state administration A sector with a central angle measure of(in radians) has a radius of 12 cm.6||6What is the area of the sector?r = 12 cm Choose the correct verbs from the word bank and conjugate them to complete the following sentences.Your options are:cantar,haber,llegar,poder,saber,1) Nos sorprende que t no _____ nadar.2) Temo que ellos no _____ a tiempo a clase.3)Andrs tiene miedo de que l _____ ir a la boda.4) Es extrao que mi hermana no _____ venido.5) Es una lstima que nosotras no _____ mejor. After one rotation of the wheel, how many inches further has the truck with the larger tires traveledthan the truck with the factory-installed (old) tires?